Mathcenter Forum  

Go Back   Mathcenter Forum > คณิตศาสตร์โอลิมปิก และอุดมศึกษา > ข้อสอบโอลิมปิก
สมัครสมาชิก คู่มือการใช้ รายชื่อสมาชิก ปฏิทิน ข้อความวันนี้

ตั้งหัวข้อใหม่ Reply
 
เครื่องมือของหัวข้อ ค้นหาในหัวข้อนี้
  #1  
Old 16 กรกฎาคม 2008, 23:11
nongtum's Avatar
nongtum nongtum ไม่อยู่ในระบบ
ผู้พิทักษ์กฎทั่วไป
 
วันที่สมัครสมาชิก: 10 เมษายน 2005
ข้อความ: 3,246
nongtum is on a distinguished road
Default 49th IMO 2008, Madrid, Spain

เครดิตข้อสอบจาก mathlinks

ข้อสอบวันแรก 16 กรกฎาคม 2008

IMO 2008, Question 1

Let $H$ be the orthocenter of an acute-angled triangle $ABC$. The circle $\Gamma_A$ centered at the midpoint of $BC$ and passing through $H$ intersects the sideline $BC$ at points $A_1$ and $A_2$. Similarly, define the points $B_1,\ B_2,\ C_1$ and $C_2$.

Prove that six points $A_1,\ A_2,\ B_1,\ B_2,\ C_1$ and $C_2$ are concyclic.

Author: Andrey Gavrilyuk, Russia

IMO 2008, Question 2

(i) If $x,\ y$ and $z$ are real numbers, different from 1, such that $xyz = 1$ prove that $$\sum \frac {x^{2}}{(x - 1)^{2}} \geq 1$$
(ii) Prove that equality case is achieved for infinitely many triples of rational numbers $x,\ y$ and $z$.

Author: Walther Janous, Austria

IMO 2008, Question 3

Prove that there are infinitely many positive integers $n$ such that $n^2+1$ has a prime divisor greater than $2n+\sqrt{2n}$.

Author: Kęstutis Česnavičius, Lithuania

ข้อสอบวันที่สอง 17 กรกฎาคม 2008

IMO 2008, Question 4

Find all functions $f: (0, \infty) \mapsto (0, \infty)$ (so $f$ is a function from the positive real numbers) such that
$$\frac {\left( f(w) \right)^2 + \left( f(x) \right)^2}{\left( f(y) \right)^2 + \left( f(z) \right)^2} = \frac {w^2 + x^2}{y^2 + z^2}$$
for all positive real numbers $w,x,y,z,$ satisfying $wx = yz$.

Author: Hojoo Lee, South Korea

IMO 2008, Question 5

Let $n$ and $k$ be positive integers with $k\ge n$ and $k-n$ an even number. Let $2n$ lamps labeled $1, 2, ...,2n$ be given, each of which can be either on or off. Initially all the lamps are off. we consider sequences of steps: at each step one of the lamps is switched (from on to off or from off to on).

Let $N$ be the number of such sequences consisting of $k$ steps and resulting in the state where lamps $1$ through $n$ are all on, and lamps $n+1$ through $2n$ are all off.

Let $M$ be number of such sequences consisting of $k$ steps, resulting in the state where lamps $1$ through $n$ are all on, and lamps $n+1$ through $2n$ are all off, but where none of the lamps $n+1$ through $2n$ is ever switched on.

Determine $N/M$.

Author: Bruno Le Floch and Ilia Smilga, France

IMO 2008, Question 6

Let $ABCD$ be a convex quadrilateral with $BA$ different from $BC$. Denote the incircles of triangles $ABC$ and $ADC$ by $k_1$ and $k_2$ respectively. Suppose that there exists a circle $k$ tangent to ray $BA$ beyond $A$ and to the ray $BC$ beyond $C$, which is also tangent to the lines $AD$ and $CD$.

Prove that the common external tangents to $k_1$ and $k_2$ intersects on $k$.

Author: Vladimir Shmarov, Russia
__________________
คนไทยร่วมใจอย่าใช้ภาษาวิบัติ
ฝึกพิมพ์สัญลักษณ์สักนิด ชีวิต(คนตอบและคนถาม)จะง่ายขึ้นเยอะ (จริงๆนะ)

Stay Hungry. Stay Foolish.

18 กรกฎาคม 2008 18:13 : ข้อความนี้ถูกแก้ไขแล้ว 3 ครั้ง, ครั้งล่าสุดโดยคุณ nongtum
เหตุผล: updated
ตอบพร้อมอ้างอิงข้อความนี้
  #2  
Old 16 กรกฎาคม 2008, 23:16
RoSe-JoKer's Avatar
RoSe-JoKer RoSe-JoKer ไม่อยู่ในระบบ
บัณฑิตฟ้า
 
วันที่สมัครสมาชิก: 25 พฤศจิกายน 2007
ข้อความ: 390
RoSe-JoKer is on a distinguished road
Default

ข้อ 2
A.ผม homogenize ไปเลย แทน $x=\frac{a}{b} y=\frac{b}{c} z=\frac{c}{a}$
อสมการที่ต้องพิสูจน์คือ
$\sum_.\frac{a^2}{(a-b)^2}\geqslant 1$
ให้
$p=\frac{a}{a-b}$
$q=\frac{b}{b-c}$
$r=\frac{c}{c-a}$
เราจะต้องพิสูจน์ว่า $p^2+q^2+r^2\geqslant 1$
แต่เห็นได้ว่า
$pqr=(p-1)(q-1)(r-1)$
ได้ว่า $\sum pq +1= \sum p$
ยกกำลังสองทั้งสองข้างได้ว่า
$(\sum pq)^2+2\sum pq +1=\sum p^2 +2\sum pq$
ดังนั้น $p^2+q^2+r^2=1+(\sum pq)^2\geqslant 1$ ...
ส่วนวิธีทำข้อ B ของผมนั้นจาก
$p^2+q^2+r^2=1+(\sum pq)^2$ ได้ว่าอสมการจะเป็นสมการก็ต่อเมื่อ
$pq+qr+rp=0$
ก็แทนค่า p,q,r กลับและจัดรูปเราจะได้ว่า
$a^2b+b^2c+c^2a=3abc$
จากตรงนี้ผมก็ให้
$a=m$
$b=m+n$ โดยที่ $m,n$ เป็นจำนวนตรรกยะ
แทนค่าลงไปในสมการได้ว่า
$0=mc^2+(-2m^2-mn+n^2)c+m^3+m^2n^2$
ก็จะได้ว่า $c=\frac{1}{2m}({2m^2+mn-n^2\pm \sqrt{m^2n^2+n^2-2mn^2-4m^2n^2} })$
ซึ่งตรงนี้ก็จะมีปัญหาคือ$\sqrt{m^2n^2+n^2-2mn^2-4m^2n}$ อาจเป็นจำนวนอตรรกยะผมก็เลยให้
$m^2n^2+n^2-2mn^2-4m^2n=k^2$ เสียโดยที่ k เป็นจำนวนตรรกยะจึงได้ว่า
$n=m+\sqrt{4m^2+k^2}$
ซึ่งผลที่ตามมาก็คือจะได้ว่า
$a=m $
$b=m+n=m+\sqrt{4m^2+k^2}$
$c=\frac{1}{2m}({2m^2+mn-n^2\pm \sqrt{m^2n^2+n^2-2mn^2-4m^2n^2} })$ (แทนค่า n ไปด้วย..แต่ผมขี้เกียจพิมพ์)
ซึ่งมีจำนวนตรรกยะ m,k เป็นอนันต์อยู่แล้วดังนั้นเราได้ว่ามีคู่อันดับ
$(\frac{a}{b},\frac{b}{c},\frac{c}{a})=(x,y,z)$ ที่เป็นจำนวนตรรกยะเป็นจำนวนอนันต์อยู่แล้ว
__________________
Rose_joker @Thailand
Serendipity

17 กรกฎาคม 2008 17:43 : ข้อความนี้ถูกแก้ไขแล้ว 4 ครั้ง, ครั้งล่าสุดโดยคุณ RoSe-JoKer
ตอบพร้อมอ้างอิงข้อความนี้
  #3  
Old 16 กรกฎาคม 2008, 23:17
Anonymous314's Avatar
Anonymous314 Anonymous314 ไม่อยู่ในระบบ
บัณฑิตฟ้า
 
วันที่สมัครสมาชิก: 16 มีนาคม 2008
ข้อความ: 546
Anonymous314 is on a distinguished road
Default

ผมว่าข้อ 3 สวยมากเลยครับ
ตอบพร้อมอ้างอิงข้อความนี้
  #4  
Old 17 กรกฎาคม 2008, 09:28
dektep's Avatar
dektep dektep ไม่อยู่ในระบบ
กระบี่ประสานใจ
 
วันที่สมัครสมาชิก: 07 มีนาคม 2007
ข้อความ: 580
dektep is on a distinguished road
Default

ผมว่าข้อสองนั้นค่อนข้างเป็นที่รู้จักพอสมควรเลยนะครับ แค่ดัดแปลงมานิดหน่อย
ตอบพร้อมอ้างอิงข้อความนี้
  #5  
Old 17 กรกฎาคม 2008, 10:22
nooonuii nooonuii ไม่อยู่ในระบบ
ผู้พิทักษ์กฎทั่วไป
 
วันที่สมัครสมาชิก: 25 พฤษภาคม 2001
ข้อความ: 6,408
nooonuii is on a distinguished road
Default

อ้างอิง:
ข้อความเดิมเขียนโดยคุณ nongtum View Post
IMO 2008, Question 2

(i) If $x,\ y$ and $z$ are real numbers, different from 1, such that $xyz = 1$ prove that $$\sum \frac {x^{2}}{(x - 1)^{2}} \geq 1$$
(ii) Prove that equality case is achieved for infinitely many triples of rational numbers $x,\ y$ and $z$.
__________________
site:mathcenter.net คำค้น
ตอบพร้อมอ้างอิงข้อความนี้
  #6  
Old 17 กรกฎาคม 2008, 10:53
Art_ninja's Avatar
Art_ninja Art_ninja ไม่อยู่ในระบบ
จอมยุทธ์หน้าหยก
 
วันที่สมัครสมาชิก: 31 มีนาคม 2007
ข้อความ: 184
Art_ninja is on a distinguished road
Smile

ผมลองคิดดูได้อีกวิธีนึงน่ะครับ เมื่อ $xyz=1$ พบว่า
$\frac {x^{2}}{(x - 1)^{2}}+\frac {y^{2}}{(y - 1)^{2}}+\frac {z^{2}}{(z - 1)^{2}}-1 \geq 0 \Leftrightarrow \frac {(xy+yz+zx-3)^{2}}{(x-1)^2(y-1)^2(z-1)^2} \geq 0$
ซึ่งอสมการจะเป็นสมการก็ต่อเมื่อ $xy+yz+zx=3$ ที่เหลือก็ไม่ยากแล้วครับ

ป.ล.ว่าแต่ปีนี้ IMO ข้อ 2 ง่ายกว่าข้อ 1 อีกนะครับ
__________________
Defeat myself successfully is the most successful in my life...

17 กรกฎาคม 2008 11:51 : ข้อความนี้ถูกแก้ไขแล้ว 2 ครั้ง, ครั้งล่าสุดโดยคุณ Art_ninja
ตอบพร้อมอ้างอิงข้อความนี้
  #7  
Old 17 กรกฎาคม 2008, 12:27
Art_ninja's Avatar
Art_ninja Art_ninja ไม่อยู่ในระบบ
จอมยุทธ์หน้าหยก
 
วันที่สมัครสมาชิก: 31 มีนาคม 2007
ข้อความ: 184
Art_ninja is on a distinguished road
Default

ข้อ $1$
จากรูปจะเห็นว่าเส้นตั้งฉากที่ลากจากจุด $B$ ไปยัง $AC$ ตั้งฉากกับ $EF$ และคอร์ดร่วมของวงกลมทั้งสองวงตั้งฉากกับ $EF$ ด้วย เนื่องจากทั้งสองเส้นได้ผ่านจุด $\rm orthocenter$ ซึ่งก็คือจุด $H$ ดังนั้นจุด $B$ อยู่บนคอร์ดร่วมของวงกลมทั้งสองวง โดย power of point จะได้ว่า $BA_1\times BA_2 = BG\times BH = BC_1 \times BC_2$
ดังนั้นจุด $A_1,A_2,C_1,C_2$ อยู่บนวงกลมเดียวกัน ทำแบบนี้กับจุด $A$ และ $C$ จะได้ว่าทั้ง $6$ จุดอยู่วงกลมเดียวกันทุกจุดครับ
Edit:จุดศูนย์กลางของวงกลมก็คือจุดศูนย์กลางของวงกลมล้อมรอบสามเหลี่ยม ABC ครับ
ดังนั้นเราก็แสดงต่อได้ว่าวงกลมทั้งสามวงมีจุดศูนย์กลางเดียวกัน ขออภัยที่ไม่ได้ระบุให้ชัดเจนด้วยนะครับ
รูปภาพที่แนบมาด้วย
 
__________________
Defeat myself successfully is the most successful in my life...

18 กรกฎาคม 2008 14:32 : ข้อความนี้ถูกแก้ไขแล้ว 1 ครั้ง, ครั้งล่าสุดโดยคุณ Art_ninja
ตอบพร้อมอ้างอิงข้อความนี้
  #8  
Old 17 กรกฎาคม 2008, 17:40
RoSe-JoKer's Avatar
RoSe-JoKer RoSe-JoKer ไม่อยู่ในระบบ
บัณฑิตฟ้า
 
วันที่สมัครสมาชิก: 25 พฤศจิกายน 2007
ข้อความ: 390
RoSe-JoKer is on a distinguished road
Default

ข้อสอบวันที่สอง update ที่ mathlinks แล้วครับ ;-)
ผมว่าข้อหนึ่งโจทย์เขาถามว่า
คุณผู้เข้าแข่งขัน คุณรู้จัก Power of the point หรือไม่
__________________
Rose_joker @Thailand
Serendipity

17 กรกฎาคม 2008 17:44 : ข้อความนี้ถูกแก้ไขแล้ว 2 ครั้ง, ครั้งล่าสุดโดยคุณ RoSe-JoKer
ตอบพร้อมอ้างอิงข้อความนี้
  #9  
Old 17 กรกฎาคม 2008, 22:00
viridae's Avatar
viridae viridae ไม่อยู่ในระบบ
เริ่มฝึกวรยุทธ์
 
วันที่สมัครสมาชิก: 09 พฤษภาคม 2008
ข้อความ: 15
viridae is on a distinguished road
Default

อ้างอิง:
ข้อความเดิมเขียนโดยคุณ nooonuii View Post
ทำไมถึงได้ว่าเศษเป็นหนึ่งอ่ะคะ ไม่เข้าใจ T^T

อ๋อ ... เข้าใจแล้วคะ แหะๆ
__________________
Demandez, et il vous sera donne; cherchez, et vous trouverez
จงร้องขอและท่านจะได้รับ จงค้นหาแล้วท่านจะพบ

18 กรกฎาคม 2008 02:24 : ข้อความนี้ถูกแก้ไขแล้ว 1 ครั้ง, ครั้งล่าสุดโดยคุณ viridae
ตอบพร้อมอ้างอิงข้อความนี้
  #10  
Old 18 กรกฎาคม 2008, 06:32
RoSe-JoKer's Avatar
RoSe-JoKer RoSe-JoKer ไม่อยู่ในระบบ
บัณฑิตฟ้า
 
วันที่สมัครสมาชิก: 25 พฤศจิกายน 2007
ข้อความ: 390
RoSe-JoKer is on a distinguished road
Default

ผมหวังว่าคุณ Seemeriast Warut_suk จะได้เหรียญทอง IMO นะครับ สถานการณ์ตอนนี้ดูเหมือนคุณ Murderous at IPST น่าจะได้เหรียญทองแน่ๆแล้วแหละครับ ส่วนคุณ dektepteptep นิเงินเหอะครับ ชอบแกล้งผมดีนัก
__________________
Rose_joker @Thailand
Serendipity
ตอบพร้อมอ้างอิงข้อความนี้
  #11  
Old 18 กรกฎาคม 2008, 10:44
Art_ninja's Avatar
Art_ninja Art_ninja ไม่อยู่ในระบบ
จอมยุทธ์หน้าหยก
 
วันที่สมัครสมาชิก: 31 มีนาคม 2007
ข้อความ: 184
Art_ninja is on a distinguished road
Default

ข้อ 4 ครับผม
แทนค่า $x=y=z=w$ จะได้ว่า $f(x^2)=(f(x))^2$
จัดรูปโจทย์ใหม่เป็น $(y^2+z^2)(f(x^2)+f(w^2))=(x^2+w^2)(f(w^2)+f(x^2))$
แทนค่า $x=w$ จะได้ว่า $yz=x^2$ ดังนั้น $(y^2+z^2)f(x^2)=(yz)(f(y^2)+f(z^2))$
แทนค่า $y$ ด้วย $1$ จะเห็นว่า $z=x^2$ ดังนั้น $(z^2+1)f(z)=(z)(f(z^2)+1) \Leftrightarrow (f(z)-z)(zf(z)-1)=0$
ดังนั้น $f(x)\equiv x$ และ $f(x)\equiv \frac{1}{x}$ เป็นคำตอบของข้อนี้ ซึ่งตรวจสอบด้วยการแทนค่าแล้วพบว่าใช้ได้จริง

อ้างอิง:
ข้อความเดิมเขียนโดยคุณ RoSe-JoKer View Post
ผมหวังว่าคุณ Seemeriast Warut_suk จะได้เหรียญทอง IMO นะครับ สถานการณ์ตอนนี้ดูเหมือน
คุณ Murderous at IPST น่าจะได้เหรียญทองแน่ๆแล้วแหละครับ ส่วนคุณ dektepteptep นิเงินเหอะครับ ชอบแกล้งผมดีนัก
แต่ผมอยากให้พี่ทุกคนได้เหรียญทองหมดนะครับ

หมายเหตุ: โจทย์ IMO ฉบับภาษาไทยออกแล้วนะครับ http://www.imo-2008.es/examenes/tha.pdf
__________________
Defeat myself successfully is the most successful in my life...

18 กรกฎาคม 2008 11:06 : ข้อความนี้ถูกแก้ไขแล้ว 1 ครั้ง, ครั้งล่าสุดโดยคุณ Art_ninja
ตอบพร้อมอ้างอิงข้อความนี้
  #12  
Old 18 กรกฎาคม 2008, 11:56
RoSe-JoKer's Avatar
RoSe-JoKer RoSe-JoKer ไม่อยู่ในระบบ
บัณฑิตฟ้า
 
วันที่สมัครสมาชิก: 25 พฤศจิกายน 2007
ข้อความ: 390
RoSe-JoKer is on a distinguished road
Default

น้อง Art_ninja ครับ Solution ข้อ 4 ของน้องยังไม่สมบูรณ์หรือป่าวครับ? ตรงตอนสรุปอะครับ
__________________
Rose_joker @Thailand
Serendipity
ตอบพร้อมอ้างอิงข้อความนี้
  #13  
Old 18 กรกฎาคม 2008, 12:16
Art_ninja's Avatar
Art_ninja Art_ninja ไม่อยู่ในระบบ
จอมยุทธ์หน้าหยก
 
วันที่สมัครสมาชิก: 31 มีนาคม 2007
ข้อความ: 184
Art_ninja is on a distinguished road
Default

อ้างอิง:
ข้อความเดิมเขียนโดยคุณ RoSe-JoKer View Post
น้อง Art_ninja ครับ Solution ข้อ 4 ของน้องยังไม่สมบูรณ์หรือป่าวครับ? ตรงตอนสรุปอะครับ
ขอบคุณครับ ไม่สมบูรณ์จริงๆครับ
เพิ่มเติม
สมมติให้มี $p,q$ ที่ไม่ใช่ $1$ ซึ่ง $f(p)=p และ f(q)=\frac{1}{q}$
แทนค่า $x\rightarrow 1,y\rightarrow p,z\rightarrow q$ แน่นอนว่า $w=pq$ ลงในสมการ
$(y+z)(f(x)+f(w))=(x+w)(f(z)+f(y))$ จะได้ว่า
$(p+q)(f(pq)+1)=(pq+1)(f(p)+f(q))\Leftrightarrow (p+q)(f(pq)+1)=(pq+1)(p+\frac{1}{q})$
ถ้า $f(pq)=pq$ จะได้ว่า $q=1$ และถ้า $f(pq)=\frac{1}{pq}$ จะได้ว่า $p=q=1$ เกิดข้อขัดแย้ง ดังนั้นฟังก์ชันที่สองคล้องมีเพียงสองฟังก์ชันที่กล่าวมาครับ

ป.ล. ไม่ทราบว่าข้อ $5$ ตอบว่า $2^{k-n}$ รึเปล่าครับ
__________________
Defeat myself successfully is the most successful in my life...

18 กรกฎาคม 2008 12:22 : ข้อความนี้ถูกแก้ไขแล้ว 1 ครั้ง, ครั้งล่าสุดโดยคุณ Art_ninja
ตอบพร้อมอ้างอิงข้อความนี้
  #14  
Old 18 กรกฎาคม 2008, 14:05
Onasdi's Avatar
Onasdi Onasdi ไม่อยู่ในระบบ
กระบี่ประสานใจ
 
วันที่สมัครสมาชิก: 12 พฤษภาคม 2005
ข้อความ: 760
Onasdi is on a distinguished road
Default

ข้อ 1 ความเห็น 7 ผมว่าต้องให้เหตุผลอีกก่อนสรุปว่า ทั้งหกจุดอยู่บนวงกลมเดียวกัน นะครับ เพราะมีจุดร่วมกันแค่สองจุด อาจจะไม่ใช่วงกลมเดียวกันก็ได้
ตอบพร้อมอ้างอิงข้อความนี้
  #15  
Old 18 กรกฎาคม 2008, 18:10
Art_ninja's Avatar
Art_ninja Art_ninja ไม่อยู่ในระบบ
จอมยุทธ์หน้าหยก
 
วันที่สมัครสมาชิก: 31 มีนาคม 2007
ข้อความ: 184
Art_ninja is on a distinguished road
Default

เติมให้แล้วนะครับ ว่าแต่ข้อ $5$ ตอบว่า $2^{k-n}$ รึเปล่าครับ ผมไม่แน่ใจ
__________________
Defeat myself successfully is the most successful in my life...
ตอบพร้อมอ้างอิงข้อความนี้
ตั้งหัวข้อใหม่ Reply


หัวข้อคล้ายคลึงกัน
หัวข้อ ผู้ตั้งหัวข้อ ห้อง คำตอบ ข้อความล่าสุด
TurkeyTsT 2008 dektep ข้อสอบโอลิมปิก 5 29 กรกฎาคม 2008 11:34
ผลการคัดเลือกผู้แทนประเทศไทยปี 2008 kanakon ข่าวคราวแวดวง ม.ปลาย 20 11 กรกฎาคม 2008 19:01
APMO 2008 dektep ข้อสอบโอลิมปิก 17 22 มิถุนายน 2008 22:20


กฎการส่งข้อความ
คุณ ไม่สามารถ ตั้งหัวข้อใหม่ได้
คุณ ไม่สามารถ ตอบหัวข้อได้
คุณ ไม่สามารถ แนบไฟล์และเอกสารได้
คุณ ไม่สามารถ แก้ไขข้อความของคุณเองได้

vB code is On
Smilies are On
[IMG] code is On
HTML code is Off
ทางลัดสู่ห้อง


เวลาที่แสดงทั้งหมด เป็นเวลาที่ประเทศไทย (GMT +7) ขณะนี้เป็นเวลา 23:15


Powered by vBulletin® Copyright ©2000 - 2024, Jelsoft Enterprises Ltd.
Modified by Jetsada Karnpracha